PT5.S1.Q13 - The policy of nuclear deterrence

Ashley2018-1Ashley2018-1 Alum Member
edited May 2021 in Logical Reasoning 2249 karma

anticipated right answer but still got it wrong :(

Premise: there have been no nuclear wars
Conclusion: nuclear deterrence has worked

I thought there might be an alternative explanation for there not being any nuclear war so I thought A provided that answer. If nukes were expensive wouldn’t that provide an alternative explanation for why there isn’t nuclear war?

Admin Note: Edited the title. Please use the format "PT#.S#.Q# - brief description of question"

Comments

  • canihazJDcanihazJD Alum Member Sage
    8318 karma

    While providing an alternative explanation could weaken, what you need is a description of why the conclusion does not follow from the premise you are given. Just bc nukes are expensive doesn't mean we can't conclude deterrence worked.

  • Ashley2018-1Ashley2018-1 Alum Member
    2249 karma

    Oh. But doesn't E say something quite similar? It brings up an economic reason as to why nuclear war hasn't broken out.

  • tahurrrrrtahurrrrr Member
    1106 karma

    The difference between A and E is that A is phrased for a weakening question. But this is a flaw question. A flaw answer choice is saying "the argument is flawed because X" whereas a weakening choice can provide a specific example that would weaken the argument (such as A).

    A actually attacks the premise while E just states what's wrong with the argument, which is what you're looking to do in a flaw question.

  • Ashley2018-1Ashley2018-1 Alum Member
    edited May 2021 2249 karma

    Well I thought A was implying there was a flaw with the argument and so went for it...

  • Ashley2018-1Ashley2018-1 Alum Member
    2249 karma

    Wait...so if this were a weakening ?, would A be correct? Eh...

  • Ashley2018-1Ashley2018-1 Alum Member
    2249 karma

    I was thinking countries didn’t have nukes because it was expensive to maintain them and so that was why there wasn’t any nuclear war

  • tahurrrrrtahurrrrr Member
    1106 karma

    This is PT5 S1 Q13 and question stem reads "Which one of the following, if true, indicates a flaw in the argument?" yes?

    This is undoubtedly a flaw question stem

    And yes, A could be a correct answer if this were a weakening question. But the question is asking about what's wrong with the argument in general, and A doesn't answer what's generally wrong with the argument.

  • Ashley2018-1Ashley2018-1 Alum Member
    2249 karma

    I had someone else tell me A was wrong because it was taking about high amount of nuclear armaments while the stimulus was referring to any level of nuclear armaments

  • Ashley2018-1Ashley2018-1 Alum Member
    2249 karma

    I misread the stem; for some reason this question was placed in the weakening section instead of the flaw section so I assumed it was a weaken question.

  • pnwrunnerpnwrunner Alum Member
    edited January 2023 100 karma

    Conclusion:
    The policy of nuclear deterrence is the sole explanation of why major powers have not used nuclear armaments (weapons) and there hasn't been a WWIII.

    Why?
    Premise a: Nuclear weapons have been in existence since WWII.
    Premise b: Major powers recognize that a worldwide nuclear war would position the world in an unusable state (uninhabitable).
    Premise c: WWIII between superpowers (major powers) hasn't happened.

    Assumptions:
    If there was a WWIII, nuclear weapons would be used.
    The policy is the sole explanation, not the fear of an unusable world.

    Pre-phrase:
    There may be other explanations than the policy of nuclear deterrence that influence either exclusively or in co-incidence.

    Notes:
    The argument supplies us with two possible (Premise b and Conclusion) explanations but concludes that one of them (Conclusion) is the reason without giving any evidence that there are not alternate explanations.

    Answer choice analysis:
    (A) Incorrect because the passage doesn't provide us with any information about how nuclear weapons contribute to an economy. If we accept this as relating to something not mentioned (flaw) in the argument, we still need to make several outside the passage assumptions to get us there. None of which are provided.

    (B) This could be true or false. We don't know. Equally, the passage only provides us with a claim that the policy of nuclear deterrence has worked up until this point. This answer choice goes beyond the present.

    (C) We aren't given any knowledge in the passage about the requirements of the policy of nuclear deterrence. This claim seems true, but it could also be false.

    (D) This answer choice provides us context and likely supports the argument.

    (E) Points out that with the evidence provided in the argument the premises leave the door open for alternate explanations.

    Admin Note: Deleted the stimulus because it is against our Forum Rules to post the LSAT questions or Answer Choices on the forum

Sign In or Register to comment.